2
$\begingroup$

Is there an integer $m\geq 1$ such that $2^m+3^m$ is a perfect power?

$\endgroup$
7
  • 3
    $\begingroup$ The question is unclear (what is quantified how) and (at least the way it is phrased) it is not clear to me that this is a research level question. Please reformulate to explain what exactly you are asking. $\endgroup$
    – Max Horn
    Jun 27, 2011 at 13:02
  • 4
    $\begingroup$ I found the question perfectly clear actually: is there an integer $m \ge 1$ such that $2^m + 3^m$ is a perfect power? A quick numerical check gives no examples for $1 \le m \le 50000$. $\endgroup$ Jun 27, 2011 at 13:08
  • 2
    $\begingroup$ David, yes, that seems like a plausible guess! But I'd still love if the original poster could clarify the question / quantization $\endgroup$
    – Max Horn
    Jun 27, 2011 at 13:13
  • 1
    $\begingroup$ For example, does the use of letter $p$ mean a prime? $\endgroup$ Jun 27, 2011 at 13:43
  • 2
    $\begingroup$ Hi i'm the poster. Sorry i stated the question in a lax way. The question is exactly as DL understand it. $\endgroup$
    – Eric
    Jun 27, 2011 at 13:45

3 Answers 3

6
$\begingroup$

If you really wanted to prove this (and I'm afraid that I'm not sure why you would), you could invoke a Theorem of Darmon and Merel for $n=2$ and $3$, check that there are no solutions with $p \leq 5$, say, and then write down the usual $(n,n,n)$ Frey curve, assuming $n \geq 5$ is prime (which leads to a weight $2$, level $6$ cuspidal newform and hence the desired contradiction).

Of course, this is an absurdly big hammer for such a problem and likely something {\it much} simpler works.

$\endgroup$
4
  • $\begingroup$ Using my notations: in that paper they assumed $p\geq 7$ is prime, NOT n (which is the exponent of $a$!). Are you sure you didn't mix the notations up? $\endgroup$
    – Eric
    Jun 27, 2011 at 16:16
  • $\begingroup$ I think Mike did not mix the notation up. What he says is that the paper by Darmon and Merel implies that there are no solutions to your equation with $n=2$ and $n=3$. He then goes on to suggest that it suffices to consider $n$ prime (this is obvious: if $n$ is not prime, say $n=q\cdot m$, then rewrite $a^n$ as $(a^m)^q$), and then proposes a solution involving a Frey curve (which I don't claim to understand, as I don't know anything about Frey curves ;). $\endgroup$
    – Max Horn
    Jun 27, 2011 at 17:14
  • $\begingroup$ The Frey curve corresponds to $$ 2^p \cdot 1^n + 3^p \cdot 1^n = a^n. $$ The ``machinery'' works for prime $n \geq 5$ in this case. Of course, it would work equally well for $$ 2^p+3^q=a^n $$ with $p \geq 4$. $\endgroup$ Jun 27, 2011 at 20:04
  • $\begingroup$ Thanks! And to quote Gjergji Zaimi:"...(this problem) would amount to proving an effective version of Zsigmondy's theorem (where there is a Zsigmondy prime of exponent 1), which is at least as hard as FLT... I believe there are no simpler approaches" $\endgroup$
    – Eric
    Jun 28, 2011 at 6:29
0
$\begingroup$

By the Fermat theorem: n is not divisible by p.

$\endgroup$
1
  • 2
    $\begingroup$ Probably the case (2,3) is far easier than Wiles. $\endgroup$ Jun 27, 2011 at 13:42
0
$\begingroup$

I was hoping to see a post by Gjergji Zaimi on this question. My guess is he deleted whatever he might have had, from which I had hoped to learn something. So I will post a start of an elementary approach in the hopes that he or someone else will finish it.

Consider the case that p is an odd positive integer. Then a must be a multiple of 5, and either n is one, or else 2^p + 3^p is a multiple of 25, in which case p must be an odd multiple of 5 by considering the sum mod 25. So in this case a^n is a multiple of 3025 if n is not 1. This can probably be refined by looking at 2^p + 3^p mod 125, and hopefully considerations mod 11 may finish it off.

Now assume p=2q for some positive integer q. If n were even we could represent 3^p by (a - 2^q)(a+2^q), which would give 3^p = 2^(q+1) + 1, which is not solvable in integers p and q. So n must be odd, a^n must be 1 mod 8 for p sufficiently large, and so must a if n is not 1. Again, more work needs to be done here. It looks plausible to me that n must be 1 if a is an integer.

Gerhard "Email Me About System Design" Paseman, 2011.06.27

$\endgroup$
4
  • 2
    $\begingroup$ I deleted a post where I had written something trivial (case p prime, misread the question). I don't think this problem can be finished by elementary methods, though. It would amount to proving an effective version of Zsigmondy's theorem (where there is a Zsigmondy prime of exponent 1), which is at least as hard as FLT... $\endgroup$ Jun 28, 2011 at 3:26
  • $\begingroup$ "It would amount to proving an effective version of Zsigmondy's theorem (where there is a Zsigmondy prime of exponent 1), which is at least as hard as FLT..." So has Mike got it right there? $\endgroup$
    – Eric
    Jun 28, 2011 at 4:09
  • $\begingroup$ Yes, I believe there are no simpler approaches... $\endgroup$ Jun 28, 2011 at 4:48
  • $\begingroup$ Thank you (as always) for your insights, Gjergji. Gerhard "Ask Me About System Design" Paseman, 2011.06.27 $\endgroup$ Jun 28, 2011 at 4:57

Your Answer

By clicking “Post Your Answer”, you agree to our terms of service and acknowledge you have read our privacy policy.

Not the answer you're looking for? Browse other questions tagged or ask your own question.